Difference between revisions of "2013 AMC 8 Problems/Problem 24"

(Problem)
(See Also These Resources...Trust Us, They Will Help!)
(25 intermediate revisions by 10 users not shown)
Line 1: Line 1:
 
==Problem==
 
==Problem==
A ball with diameter 4 inches starts at point A to roll along the track shown. The track is comprised of 3 semicircular arcs whose radii are <math>R_1 = 100</math> inches, <math>R_2 = 60</math> inches, and <math>R_3 = 80</math> inches, respectively. The ball always remains in contact with the track and does not slip. What is the distance the center of the ball travels over the course from A to B?
+
Squares <math>ABCD</math>, <math>EFGH</math>, and <math>GHIJ</math> are equal in area. Points <math>C</math> and <math>D</math> are the midpoints of sides <math>IH</math> and <math>HE</math>, respectively. What is the ratio of the area of the shaded pentagon <math>AJICB</math> to the sum of the areas of the three squares?
 +
 
 +
<math> \textbf{(A)}\hspace{.05in}\frac{1}{4}\qquad\textbf{(B)}\hspace{.05in}\frac{7}{24}\qquad\textbf{(C)}\hspace{.05in}\frac{1}{3}\qquad\textbf{(D)}\hspace{.05in}\frac{3}{8}\qquad\textbf{(E)}\hspace{.05in}\frac{5}{12}</math>
 +
 
 
<asy>
 
<asy>
size(8cm);
+
pair A,B,C,D,E,F,G,H,I,J;
draw((0,0)--(480,0),linetype("3 4"));
+
 
filldraw(circle((8,0),8),black);
+
A = (0.5,2);
draw((0,0)..(100,-100)..(200,0));
+
B = (1.5,2);
draw((200,0)..(260,60)..(320,0));
+
C = (1.5,1);
draw((320,0)..(400,-80)..(480,0));
+
D = (0.5,1);
draw((100,0)--(150,-50sqrt(3)),Arrow(size=4));
+
E = (0,1);
draw((260,0)--(290,30sqrt(3)),Arrow(size=4));
+
F = (0,0);
draw((400,0)--(440,-40sqrt(3)),Arrow(size=4));
+
G = (1,0);
 +
H = (1,1);
 +
I = (2,1);
 +
J = (2,0);
 +
draw(A--B);  
 +
draw(C--B);
 +
draw(D--A)
 +
draw(F--E);  
 +
draw(I--J);
 +
draw(J--F);
 +
draw(G--H);  
 +
draw(A--J);
 +
filldraw(A--B--C--I--J--cycle,grey);
 +
draw(E--I);
 +
dot("$A$", A, NW);
 +
dot("$B$", B, NE);
 +
dot("$C$", C, NE);
 +
dot("$D$", D, NW);
 +
dot("$E$", E, NW);
 +
dot("$F$", F, SW);
 +
dot("$G$", G, S);
 +
dot("$H$", H, N);
 +
dot("$I$", I, NE);
 +
dot("$J$", J, SE);
 
</asy>
 
</asy>
  
<math>\textbf{(A)}\ 238\pi \qquad \textbf{(B)}\ 240\pi \qquad \textbf{(C)}\ 260\pi \qquad \textbf{(D)}\ 280\pi \qquad \textbf{(E)}\ 500\pi</math>
+
==Easiest Solution==
 +
 
 +
We can see that the Pentagon is made of two congruent shapes. We can fit one triangle into the gap in the upper square. Therefore, the answer is just <math>\frac{1}{3}\implies\boxed{C}</math>
 +
 
 +
==Solution 1==
 +
<asy>
 +
pair A,B,C,D,E,F,G,H,I,J,X;
 +
A = (0.5,2);
 +
B = (1.5,2);
 +
C = (1.5,1);
 +
D = (0.5,1);
 +
E = (0,1);
 +
F = (0,0);
 +
G = (1,0);
 +
H = (1,1);
 +
I = (2,1);
 +
J = (2,0);
 +
X= extension(I,J,A,B);
 +
dot(X,red);
 +
draw(I--X--B,red);
 +
draw(A--B);
 +
draw(C--B);
 +
draw(D--A);
 +
draw(F--E);
 +
draw(I--J);
 +
draw(J--F);
 +
draw(G--H);
 +
draw(A--J);
 +
filldraw(A--B--C--I--J--cycle,grey);
 +
draw(E--I);
 +
dot("$A$", A, NW);
 +
dot("$B$", B, NE);
 +
dot("$C$", C, NE);
 +
dot("$D$", D, NW);
 +
dot("$E$", E, NW);
 +
dot("$F$", F, SW);
 +
dot("$G$", G, S);
 +
dot("$H$", H, N);
 +
dot("$I$", I, NE);
 +
label("$X$", X,SE);
 +
dot("$J$", J, SE);</asy>
 +
 
 +
 
 +
First let <math>s=2</math> (where <math>s</math> is the side length of the squares) for simplicity. We can extend <math>\overline{IJ}</math> until it hits the extension of <math>\overline{AB}</math>. Call this point <math>X</math>. The area of triangle <math>AXJ</math> then is <math>\dfrac{3 \cdot 4}{2}</math> The area of rectangle <math>BXIC</math> is <math>2 \cdot 1 = 2</math>. Thus, our desired area is <math>6-2 = 4</math>. Now, the ratio of the shaded area to the combined area of the three squares is <math>\frac{4}{3\cdot 2^2} = \boxed{\textbf{(C)}\hspace{.05in}\frac{1}{3}}</math>.
 +
 
 +
==Solution 2==
 +
 
 +
<asy>
 +
pair A,B,C,D,E,F,G,H,I,J,X;
 +
A = (0.5,2);
 +
B = (1.5,2);
 +
C = (1.5,1);
 +
D = (0.5,1);
 +
E = (0,1);
 +
F = (0,0);
 +
G = (1,0);
 +
H = (1,1);
 +
I = (2,1);
 +
J = (2,0);
 +
X= (1.25,1);
 +
draw(A--B);
 +
draw(C--B);
 +
draw(D--A);
 +
draw(F--E);
 +
draw(I--J);
 +
draw(J--F);
 +
draw(G--H);
 +
draw(A--J);
 +
filldraw(A--B--C--I--J--cycle,grey);
 +
draw(E--I);
 +
dot(X,red);
 +
label("$A$", A, NW);
 +
label("$B$", B, NE);
 +
label("$C$", C, NE);
 +
label("$D$", D, NW);
 +
label("$E$", E, NW);
 +
label("$F$", F, SW);
 +
label("$G$", G, S);
 +
label("$H$", H, N);
 +
label("$I$", I, NE);
 +
label("$X$", X,SW,red);
 +
label("$J$", J, SE);</asy>
 +
 
 +
Let the side length of each square be <math>1</math>.
 +
 
 +
Let the intersection of <math>AJ</math> and <math>EI</math> be <math>X</math>.
 +
 
 +
Since <math>[ABCD]=[GHIJ]</math>, <math>AD=IJ</math>. Since <math>\angle IXJ</math> and <math>\angle AXD</math> are vertical angles, they are congruent. We also have <math>\angle JIH\cong\angle ADC</math> by definition.
 +
 
 +
So we have <math>\triangle ADX\cong\triangle JIX</math> by <math>\textit{AAS}</math> congruence. Therefore, <math>DX=JX</math>.
 +
 
 +
Since <math>C</math> and <math>D</math> are midpoints of sides, <math>DH=CJ=\dfrac{1}{2}</math>. This combined with <math>DX=JX</math> yields <math>HX=CX=\dfrac{1}{2}\times \dfrac{1}{2}=\dfrac{1}{4}</math>.
 +
 
 +
The area of trapezoid <math>ABCX</math> is <math>\dfrac{1}{2}(AB+CX)(BC)=\dfrac{1}{2}\times \dfrac{5}{4}\times 1=\dfrac{5}{8}</math>.
 +
 
 +
The area of triangle <math>JIX</math> is <math>\dfrac{1}{2}\times XJ\times IJ=\dfrac{1}{2}\times \dfrac{3}{4}\times 1=\dfrac{3}{8}</math>.
 +
 
 +
So the area of the pentagon <math>AJICB</math> is <math>\dfrac{3}{8}+\dfrac{5}{8}=1</math>.
 +
 
 +
The area of the <math>3</math> squares is <math>1\times 3=3</math>.
 +
 
 +
Therefore, <math>\dfrac{[AJICB]}{[ABCIJFED]}= \boxed{\textbf{(C)}\hspace{.05in}\frac{1}{3}}</math>.
 +
 
 +
==Solution 3==
 +
 
 +
<asy>
 +
pair A,B,C,D,E,F,G,H,I,J,K;
 +
A = (0.5,2);
 +
B = (1.5,2);
 +
C = (1.5,1);
 +
D = (0.5,1);
 +
E = (0,1);
 +
F = (0,0);
 +
G = (1,0);
 +
H = (1,1);
 +
I = (2,1);
 +
J = (2,0);
 +
K= (1.25,1);
 +
draw(A--B);
 +
draw(C--B);
 +
draw(D--A);
 +
draw(F--E);
 +
draw(I--J);
 +
draw(J--F);
 +
draw(G--H);
 +
draw(A--J);
 +
filldraw(A--B--C--I--J--cycle,grey);
 +
draw(E--I);
 +
dot(K,red);
 +
label("$A$", A, NW);
 +
label("$B$", B, NE);
 +
label("$C$", C, NE);
 +
label("$D$", D, NW);
 +
label("$E$", E, NW);
 +
label("$F$", F, SW);
 +
label("$G$", G, S);
 +
label("$H$", H, N);
 +
label("$I$", I, NE);
 +
label("$K$", K,SW,red);
 +
label("$J$", J, SE);</asy>
 +
 
 +
Let the intersection of <math>AJ</math> and <math>EI</math> be <math>K</math>.
 +
 
 +
Now we have <math>\triangle ADK</math> and <math>\triangle KIJ</math>.
 +
 
 +
Because both triangles has a side on congruent squares therefore <math>AD \cong IJ</math>.
 +
 
 +
Because <math>\angle AKD</math> and <math>\angle JKI</math> are vertical angles <math>\angle AKD \cong \angle JKI</math>.
 +
 
 +
Also both <math>\angle ADK</math> and <math>\angle JIK</math> are right angles so <math>\angle ADK \cong \angle JIK</math>.
 +
 
 +
Therefore by AAS(Angle, Angle, Side) <math>\triangle ADK \cong \triangle KIJ</math>.
 +
 
 +
Then translating/rotating the shaded <math>\triangle JIK</math> into the position of <math>\triangle ADK</math>
 +
 
 +
So the shaded area now completely covers the square <math>ABCD</math>
 +
 
 +
Set the area of a square as <math>x</math>
 +
 
 +
Therefore, <math>\frac{x}{3x}= \boxed{\textbf{(C)}\hspace{.05in}\frac{1}{3}}</math>.
  
==Solution==
+
==Solution 4==
 +
Given the information in the problem, we see that the black area is congruent to ADHGJ. Since it is half of 2/3, it takes up 1/3 of the area.
  
 
==See Also==
 
==See Also==
 
{{AMC8 box|year=2013|num-b=23|num-a=25}}
 
{{AMC8 box|year=2013|num-b=23|num-a=25}}
 
{{MAA Notice}}
 
{{MAA Notice}}

Revision as of 23:41, 15 October 2020

Problem

Squares $ABCD$, $EFGH$, and $GHIJ$ are equal in area. Points $C$ and $D$ are the midpoints of sides $IH$ and $HE$, respectively. What is the ratio of the area of the shaded pentagon $AJICB$ to the sum of the areas of the three squares?

$\textbf{(A)}\hspace{.05in}\frac{1}{4}\qquad\textbf{(B)}\hspace{.05in}\frac{7}{24}\qquad\textbf{(C)}\hspace{.05in}\frac{1}{3}\qquad\textbf{(D)}\hspace{.05in}\frac{3}{8}\qquad\textbf{(E)}\hspace{.05in}\frac{5}{12}$

[asy] pair A,B,C,D,E,F,G,H,I,J;  A = (0.5,2); B = (1.5,2); C = (1.5,1); D = (0.5,1); E = (0,1); F = (0,0); G = (1,0); H = (1,1); I = (2,1); J = (2,0);  draw(A--B);  draw(C--B);  draw(D--A);   draw(F--E);  draw(I--J);  draw(J--F);  draw(G--H);  draw(A--J);  filldraw(A--B--C--I--J--cycle,grey); draw(E--I); dot("$A$", A, NW); dot("$B$", B, NE); dot("$C$", C, NE); dot("$D$", D, NW); dot("$E$", E, NW); dot("$F$", F, SW); dot("$G$", G, S); dot("$H$", H, N); dot("$I$", I, NE); dot("$J$", J, SE); [/asy]

Easiest Solution

We can see that the Pentagon is made of two congruent shapes. We can fit one triangle into the gap in the upper square. Therefore, the answer is just $\frac{1}{3}\implies\boxed{C}$

Solution 1

[asy] pair A,B,C,D,E,F,G,H,I,J,X; A = (0.5,2); B = (1.5,2); C = (1.5,1); D = (0.5,1); E = (0,1); F = (0,0); G = (1,0); H = (1,1); I = (2,1); J = (2,0);  X= extension(I,J,A,B); dot(X,red); draw(I--X--B,red); draw(A--B);  draw(C--B);  draw(D--A);  draw(F--E);  draw(I--J);  draw(J--F);  draw(G--H);  draw(A--J);  filldraw(A--B--C--I--J--cycle,grey); draw(E--I); dot("$A$", A, NW); dot("$B$", B, NE); dot("$C$", C, NE); dot("$D$", D, NW); dot("$E$", E, NW); dot("$F$", F, SW); dot("$G$", G, S); dot("$H$", H, N); dot("$I$", I, NE); label("$X$", X,SE); dot("$J$", J, SE);[/asy]


First let $s=2$ (where $s$ is the side length of the squares) for simplicity. We can extend $\overline{IJ}$ until it hits the extension of $\overline{AB}$. Call this point $X$. The area of triangle $AXJ$ then is $\dfrac{3 \cdot 4}{2}$ The area of rectangle $BXIC$ is $2 \cdot 1 = 2$. Thus, our desired area is $6-2 = 4$. Now, the ratio of the shaded area to the combined area of the three squares is $\frac{4}{3\cdot 2^2} = \boxed{\textbf{(C)}\hspace{.05in}\frac{1}{3}}$.

Solution 2

[asy] pair A,B,C,D,E,F,G,H,I,J,X; A = (0.5,2); B = (1.5,2); C = (1.5,1); D = (0.5,1); E = (0,1); F = (0,0); G = (1,0); H = (1,1); I = (2,1); J = (2,0);  X= (1.25,1); draw(A--B);  draw(C--B);  draw(D--A);  draw(F--E);  draw(I--J);  draw(J--F);  draw(G--H);  draw(A--J);  filldraw(A--B--C--I--J--cycle,grey); draw(E--I); dot(X,red); label("$A$", A, NW); label("$B$", B, NE); label("$C$", C, NE); label("$D$", D, NW); label("$E$", E, NW); label("$F$", F, SW); label("$G$", G, S); label("$H$", H, N); label("$I$", I, NE); label("$X$", X,SW,red); label("$J$", J, SE);[/asy]

Let the side length of each square be $1$.

Let the intersection of $AJ$ and $EI$ be $X$.

Since $[ABCD]=[GHIJ]$, $AD=IJ$. Since $\angle IXJ$ and $\angle AXD$ are vertical angles, they are congruent. We also have $\angle JIH\cong\angle ADC$ by definition.

So we have $\triangle ADX\cong\triangle JIX$ by $\textit{AAS}$ congruence. Therefore, $DX=JX$.

Since $C$ and $D$ are midpoints of sides, $DH=CJ=\dfrac{1}{2}$. This combined with $DX=JX$ yields $HX=CX=\dfrac{1}{2}\times \dfrac{1}{2}=\dfrac{1}{4}$.

The area of trapezoid $ABCX$ is $\dfrac{1}{2}(AB+CX)(BC)=\dfrac{1}{2}\times \dfrac{5}{4}\times 1=\dfrac{5}{8}$.

The area of triangle $JIX$ is $\dfrac{1}{2}\times XJ\times IJ=\dfrac{1}{2}\times \dfrac{3}{4}\times 1=\dfrac{3}{8}$.

So the area of the pentagon $AJICB$ is $\dfrac{3}{8}+\dfrac{5}{8}=1$.

The area of the $3$ squares is $1\times 3=3$.

Therefore, $\dfrac{[AJICB]}{[ABCIJFED]}= \boxed{\textbf{(C)}\hspace{.05in}\frac{1}{3}}$.

Solution 3

[asy] pair A,B,C,D,E,F,G,H,I,J,K; A = (0.5,2); B = (1.5,2); C = (1.5,1); D = (0.5,1); E = (0,1); F = (0,0); G = (1,0); H = (1,1); I = (2,1); J = (2,0);  K= (1.25,1); draw(A--B);  draw(C--B);  draw(D--A);  draw(F--E);  draw(I--J);  draw(J--F);  draw(G--H);  draw(A--J);  filldraw(A--B--C--I--J--cycle,grey); draw(E--I); dot(K,red); label("$A$", A, NW); label("$B$", B, NE); label("$C$", C, NE); label("$D$", D, NW); label("$E$", E, NW); label("$F$", F, SW); label("$G$", G, S); label("$H$", H, N); label("$I$", I, NE); label("$K$", K,SW,red); label("$J$", J, SE);[/asy]

Let the intersection of $AJ$ and $EI$ be $K$.

Now we have $\triangle ADK$ and $\triangle KIJ$.

Because both triangles has a side on congruent squares therefore $AD \cong IJ$.

Because $\angle AKD$ and $\angle JKI$ are vertical angles $\angle AKD \cong \angle JKI$.

Also both $\angle ADK$ and $\angle JIK$ are right angles so $\angle ADK \cong \angle JIK$.

Therefore by AAS(Angle, Angle, Side) $\triangle ADK \cong \triangle KIJ$.

Then translating/rotating the shaded $\triangle JIK$ into the position of $\triangle ADK$

So the shaded area now completely covers the square $ABCD$

Set the area of a square as $x$

Therefore, $\frac{x}{3x}= \boxed{\textbf{(C)}\hspace{.05in}\frac{1}{3}}$.

Solution 4

Given the information in the problem, we see that the black area is congruent to ADHGJ. Since it is half of 2/3, it takes up 1/3 of the area.

See Also

2013 AMC 8 (ProblemsAnswer KeyResources)
Preceded by
Problem 23
Followed by
Problem 25
1 2 3 4 5 6 7 8 9 10 11 12 13 14 15 16 17 18 19 20 21 22 23 24 25
All AJHSME/AMC 8 Problems and Solutions

The problems on this page are copyrighted by the Mathematical Association of America's American Mathematics Competitions. AMC logo.png